(4 intermediate revisions by 2 users not shown)
Line 4: Line 4:
 
[[Category:problem solving]]
 
[[Category:problem solving]]
 
[[Category:random variables]]
 
[[Category:random variables]]
 +
[[Category:probability]]
  
= [[ECE_PhD_Qualifying_Exams|ECE Ph.D. Qualifying Exam]]: COMMUNICATIONS, NETWORKING, SIGNAL AND IMAGE PROESSING (CS)- Question 1, August 2001=
+
<center>
 +
<font size= 4>
 +
[[ECE_PhD_Qualifying_Exams|ECE Ph.D. Qualifying Exam]]
 +
</font size>
 +
 
 +
<font size= 4>
 +
Communication, Networking, Signal and Image Processing (CS)
 +
 
 +
Question 1: Probability and Random Processes
 +
</font size>
 +
 
 +
August 2001
 +
</center>
 
----
 
----
==Question==
 
Write it here
 
 
----
 
----
=Solution 1 (retrived from [[ |here]])=
+
'''Part 1. '''
  
 +
'''1. (10 Points)'''
  
 +
Consider the following random experiment: A fair coin is repeatedly tossed until the same outcome (H or T) appears twice in a row.
 +
 +
(a) What is the probability that this experiment terminates on or before the seventh coin toss?
 +
 +
(b) What is the probability that this experiment terminates with an even number of coin tosses?
 +
 +
:'''Click [[ECE_PhD_QE_CNSIP_2001_Problem1.1|here]] to view student [[ECE_PhD_QE_CNSIP_2001_Problem1.1|answers and discussions]]'''
 
----
 
----
==Solution 2==
+
'''Part 2.'''
Write it here.
+
 
 +
'''2. (25 Points)'''
 +
 
 +
Let <math class="inline">\mathbf{X}</math>  and <math class="inline">\mathbf{Y}</math>  be independent Poisson random variables with mean <math class="inline">\lambda</math>  and <math class="inline">\mu</math> , respectively. Let <math class="inline">\mathbf{Z}</math>  be a new random variable defined as <math class="inline">\mathbf{Z}=\mathbf{X}+\mathbf{Y}.</math>
 +
 
 +
'''(a)''' Find the probability mass function (pmf) of <math class="inline">\mathbf{Z}</math> .
 +
 
 +
'''(b)''' Find the conditional probability mass function (pmf) of <math class="inline">\mathbf{X}</math>  conditional on the event <math class="inline">\left\{ \mathbf{Z}=n\right\}</math>  . Identify the type of pmf that this is, and fully specify its parameters.
 +
 
 +
:'''Click [[ECE_PhD_QE_CNSIP_2001_Problem1.2|here]] to view student [[ECE_PhD_QE_CNSIP_2001_Problem1.2|answers and discussions]]'''
 +
----
 +
'''Part 3.'''
 +
 
 +
'''3. (30 Points)'''
 +
 
 +
Let <math class="inline">\mathbf{X}_{1},\cdots,\mathbf{X}_{n},\cdots</math>  be a sequence of random variables that are not necessarily statistically independent, but that each have identical mean <math class="inline">\mu</math>  and variance <math class="inline">\sigma^{2}</math> . Let <math class="inline">\mathbf{Y}_{1},\cdots,\mathbf{Y}_{n},\cdots</math>  be a sequence of random variable with <math class="inline">\mathbf{Y}_{n}=\frac{1}{n}\sum_{k=1}^{n}\mathbf{X}_{k}.</math>
 +
 
 +
'''(a)''' Given that <math class="inline">\mathbf{X}_{1},\cdots,\mathbf{X}_{n},\cdots</math>  are uncorrelated, determine whether or not <math class="inline">\left\{ \mathbf{Y}_{n}\right\}</math>  converges to <math class="inline">\mu</math>  in the mean square sense.
 +
 
 +
(b) Given that the covariance between <math class="inline">\mathbf{X}_{j}</math>  and <math class="inline">\mathbf{X}_{k}</math>  is given by
 +
<br>
 +
<math class="inline">cov\left(\mathbf{X}_{j},\mathbf{X}_{k}\right)=\begin{cases}
 +
\begin{array}{lll}
 +
\sigma^{2}    \text{, for }j=k\\
 +
r\sigma^{2}    \text{, for }\left|j-k\right|=1\\
 +
0    \text{, elsewhere, }
 +
\end{array}\end{cases}</math>
 +
<br>
 +
where <math class="inline">-1\leq r\leq1</math> , determine whether or not <math class="inline">\left\{ \mathbf{Y}_{n}\right\}</math>  converges to <math class="inline">\mu</math>  in the mean square sense.
 +
 
 +
:'''Click [[ECE_PhD_QE_CNSIP_2001_Problem1.3|here]] to view student [[ECE_PhD_QE_CNSIP_2001_Problem1.3|answers and discussions]]'''
 +
----
 +
'''Part 4.'''
 +
 
 +
4. (35 Points)
 +
 
 +
Let <math class="inline">\left\{ t_{k}\right\}</math>  be the set of Poisson points corresponding to a homogeneous Poisson process with parameters <math class="inline">\lambda</math>  on the real line such that if <math class="inline">\mathbf{N}\left(t_{1},t_{2}\right)</math>  is defined as the number of points in the interval <math class="inline">\left[t_{1},t_{2}\right)</math> , then <math class="inline">P\left(\left\{ N\left(t_{1},t_{2}\right)=k\right\} \right)=\frac{\left[\lambda\left(t_{2}-t_{1}\right)\right]^{k}e^{-\lambda\left(t_{2}-t_{1}\right)}}{k!}\;,\qquad k=0,1,2,\cdots,\; t_{2}>t_{1}\geq0. Let \mathbf{X}\left(t\right)=\mathbf{N}\left(0,t\right)</math>  be the Poisson counting process for <math class="inline">t>0</math>  (note that <math class="inline">\mathbf{X}\left(0\right)=0</math> ).
 +
 
 +
(a) Find the (first order) characteristic function of <math class="inline">\mathbf{X}\left(t\right)</math> .
 +
 
 +
(b) Find the mean and variance of <math class="inline">\mathbf{X}\left(t\right)</math> .
 +
 
 +
(c) Deriven an expression for the autocorrelation function of <math class="inline">\mathbf{X}\left(t\right)</math> .
 +
 
 +
(d) Assuming that <math class="inline">t_{2}>t_{1}</math> , find an expression for <math class="inline">P\left(\left\{ \mathbf{X}\left(t_{1}\right)=m\right\} \cap\left\{ \mathbf{X}\left(t_{2}\right)=n\right\} \right)</math> , for all <math class="inline">m=0,1,2,\cdots</math>  and <math class="inline">n=0,1,2,\cdots</math> .
 +
 
 +
:'''Click [[ECE_PhD_QE_CNSIP_2001_Problem1.4|here]] to view student [[ECE_PhD_QE_CNSIP_2001_Problem1.4|answers and discussions]]'''
 
----
 
----
 
[[ECE_PhD_Qualifying_Exams|Back to ECE Qualifying Exams (QE) page]]
 
[[ECE_PhD_Qualifying_Exams|Back to ECE Qualifying Exams (QE) page]]

Latest revision as of 01:25, 9 March 2015


ECE Ph.D. Qualifying Exam

Communication, Networking, Signal and Image Processing (CS)

Question 1: Probability and Random Processes

August 2001



Part 1.

1. (10 Points)

Consider the following random experiment: A fair coin is repeatedly tossed until the same outcome (H or T) appears twice in a row.

(a) What is the probability that this experiment terminates on or before the seventh coin toss?

(b) What is the probability that this experiment terminates with an even number of coin tosses?

Click here to view student answers and discussions

Part 2.

2. (25 Points)

Let $ \mathbf{X} $ and $ \mathbf{Y} $ be independent Poisson random variables with mean $ \lambda $ and $ \mu $ , respectively. Let $ \mathbf{Z} $ be a new random variable defined as $ \mathbf{Z}=\mathbf{X}+\mathbf{Y}. $

(a) Find the probability mass function (pmf) of $ \mathbf{Z} $ .

(b) Find the conditional probability mass function (pmf) of $ \mathbf{X} $ conditional on the event $ \left\{ \mathbf{Z}=n\right\} $ . Identify the type of pmf that this is, and fully specify its parameters.

Click here to view student answers and discussions

Part 3.

3. (30 Points)

Let $ \mathbf{X}_{1},\cdots,\mathbf{X}_{n},\cdots $ be a sequence of random variables that are not necessarily statistically independent, but that each have identical mean $ \mu $ and variance $ \sigma^{2} $ . Let $ \mathbf{Y}_{1},\cdots,\mathbf{Y}_{n},\cdots $ be a sequence of random variable with $ \mathbf{Y}_{n}=\frac{1}{n}\sum_{k=1}^{n}\mathbf{X}_{k}. $

(a) Given that $ \mathbf{X}_{1},\cdots,\mathbf{X}_{n},\cdots $ are uncorrelated, determine whether or not $ \left\{ \mathbf{Y}_{n}\right\} $ converges to $ \mu $ in the mean square sense.

(b) Given that the covariance between $ \mathbf{X}_{j} $ and $ \mathbf{X}_{k} $ is given by
$ cov\left(\mathbf{X}_{j},\mathbf{X}_{k}\right)=\begin{cases} \begin{array}{lll} \sigma^{2} \text{, for }j=k\\ r\sigma^{2} \text{, for }\left|j-k\right|=1\\ 0 \text{, elsewhere, } \end{array}\end{cases} $
where $ -1\leq r\leq1 $ , determine whether or not $ \left\{ \mathbf{Y}_{n}\right\} $ converges to $ \mu $ in the mean square sense.

Click here to view student answers and discussions

Part 4.

4. (35 Points)

Let $ \left\{ t_{k}\right\} $ be the set of Poisson points corresponding to a homogeneous Poisson process with parameters $ \lambda $ on the real line such that if $ \mathbf{N}\left(t_{1},t_{2}\right) $ is defined as the number of points in the interval $ \left[t_{1},t_{2}\right) $ , then $ P\left(\left\{ N\left(t_{1},t_{2}\right)=k\right\} \right)=\frac{\left[\lambda\left(t_{2}-t_{1}\right)\right]^{k}e^{-\lambda\left(t_{2}-t_{1}\right)}}{k!}\;,\qquad k=0,1,2,\cdots,\; t_{2}>t_{1}\geq0. Let \mathbf{X}\left(t\right)=\mathbf{N}\left(0,t\right) $ be the Poisson counting process for $ t>0 $ (note that $ \mathbf{X}\left(0\right)=0 $ ).

(a) Find the (first order) characteristic function of $ \mathbf{X}\left(t\right) $ .

(b) Find the mean and variance of $ \mathbf{X}\left(t\right) $ .

(c) Deriven an expression for the autocorrelation function of $ \mathbf{X}\left(t\right) $ .

(d) Assuming that $ t_{2}>t_{1} $ , find an expression for $ P\left(\left\{ \mathbf{X}\left(t_{1}\right)=m\right\} \cap\left\{ \mathbf{X}\left(t_{2}\right)=n\right\} \right) $ , for all $ m=0,1,2,\cdots $ and $ n=0,1,2,\cdots $ .

Click here to view student answers and discussions

Back to ECE Qualifying Exams (QE) page

Alumni Liaison

BSEE 2004, current Ph.D. student researching signal and image processing.

Landis Huffman